Вы находитесь на странице: 1из 14

U NIVERSIDADE DE B RASLIA , FACULDADE G AMA

Sinais e Sistemas - Lista 2

4 de outubro de 2015

1. Um sinal peridico de tempo contnuo x(t) tem valor real e perodo fundamental T = 8.
Os coeficientes diferentes de zero da srie de fourier de x(t) so:

a 1 = a 1 = 2, a 3 = a 3 = 4j

Expresse x(t) na forma:



A k cos(k t + k )
X
x(t ) =
k=0

Resposta:

x(t ) = a 1 e (2/T )t + a 1 e j (2/T )t + a 3 e j 3(2/T )t + a 3 e j 3(2/T )t


= 2e j (2/8)t + 2e j (2/8)t + 4 j e j 3(2/8)t 4 j e j 3(2/8)t
= 4cos( 4 t ) 8si n( 68 t)
= 4cos( 4 t ) + 8cos( 3 4 t + 2)

2. Para o sinal peridico de tempo contnuo



2 5
x(t ) = 2 + cos t + 4sen t ,
3 3

Determine a frequncia fundamental 0 e os coeficientes da srie de Fourier a k tais


que

a k e j k0 t
X
x(t ) =
k=

Resposta:

1
x(t ) = 2 + 12 e j (2/3)t + 12 e j (2/3)t 2 j e j (5/3)t + 2 j e j (5/3)t
= 2 + 12 e j 2(2/6)t + 12 e j 2(2/6)t 2 j e j 5(2/6)t + 2 j e j 5(2/6)t

A partir disso, conclui-se que a frequencia fundamental de x(t) 2/6 = /3. E os coe-
ficientes no nulos de fourier de x(t) so:
1
a 0 = 2, a 2 = a 2 = , a 5 = a 5 = 2 j
2

3. Use a equao:
1 1
Z Z
j k0 t
ak = x(t )e dt = x(t )e j k(2/T )t d t
T T T T

Para calcular os coeficientes a k para o sinal peridico de tempo contnuo


1.5, 0 t < 1
x(t ) =
1.5, 1 t < 2
com frequncia 0 = .
Resposta:

0 = , T = 2/0 = 2
R2
a k = 21 0 x(t )e j kt d t
R1 R2
a 0 = 12 0 1.5d t 12 1 1.5d t = 0
e para k 6= 0
R1 R2
a k = 12 0 1.5e j kt d t 12 1 1.5e j kt d t
3
= 2k j [1 e j k]
= 3 j k(/2)
k e si n( k
2 )

Quando k=0,
a k = T1 <T > x(t )d t = 2
R
T

(
2/T, k =0
ak = bk
j (2/T )k , k 6= 0

4. Considere um sistema LTI de tempo contnuo cuja resposta em frequncia :


Z
sen(4)
H ( j ) = h(t )e j t d t =

Se a entrada desse sistema um sinal peridico

1, 0 t < 4
x(t ) =
1, 4 t < 8

com periodo T = 8, determine a sada correspondente do sistema y(t).

2
Resposta:

x(t) real e impar, a k puramente imaginario e impar, logo a 0 = 0


R8
a k = 81 0 x(t )e j (2/8)kt d t
R4 R8
= 18 0 e j (2/8)kt d t 18 4 e j (2/8)kt d t
1
= j k [1 e j k ]
(
0, k = 0, 2, 4...
ak = 2
k = 1, 3, 5, ...
j k ,
Quando x(t) passado atraves do sistema LIT com frequencia H ( j ), a saida y(t) dada
por:
y(t ) = a H ( j k0 )e j k0 t
P
k= k

Onde 0 = 2 T = 4
a k no nulo somente para valores mpares, desta maneira:
H ( j k0 ) = H ( j k(/4)) = sik(/4)
n(k)

sempre zero para valores impares de k. e:


y(t ) = 0

5. Considere um sistema LIT causal implementado como circuito RLC mostrado na figura
abaixo. Nesse circuito, x(t) a tenso de entrada. A tenso y(t) no capacitor conside-
rada a sada do sistema.

a) Encontre a equao diferencial relacionando x(t) e y(t).


b) Determine a resposta em frequncia desse sistema considerando a sada do sis-
tema para as entradas da forma x(t ) = e j t .
c) Determine a sada y(t ) se x(t ) = sen(t ).

Resposta:

a)
d y(t )
ic = C d t
d y(t )
i R = RCdt
d 2 y(t )
VL = LC d t 2
Tenso de entrada = VR + VL + VC

3
d 2 y(t ) d y(t )
x(t ) = LC d t 2 + RC d t + y(t )
Substituindo os valores:
d 2 y(t ) d y(t )
dt2
+ d t + y(t ) = x(t )

b)
1
H ( j ) = 2 + j +1

c)
x(t) periodico com periodo 2, x(t) pode ser expresso na forma:
x(t ) = 21j e j (2/2)t 21j e j (2/2)t
os coeficientes no nulos de fourier de x(t) so

a 1 = a 1 = 21j
y(t ) = a 1 H ( j )e j t a 1 H ( j )e j t
= (1/2 j )( 1j e j t 1j e j t )
= (1/2)(e j t + e j t )
= cos(t )

6. Determine a srie de Fourier dos sinais abaixo.



1, T /2 < t < 0
a) f (t ) =
1, 0 < t < T /2

0, < t < 0
b) f (t ) = 1
t, 0<t <

c) N [n] = l = [n l N ]
P+

d) x[n] = +
P
k=
p[n kN ], para N=4,
p[n] = [n + 1] + [n 1]

e) x[n] = +
P
k=
p[n kN ], para N=10,
p[n] = [n + 1] + [n] + [n 1]

Resposta:

a)
a0 = 0
R0 R T /2
a n = T2 T /2 1cos(n0 t )d t + 0 1cos(n0 t )d t
an = 0
R0 R T /2
b n = T2 ( T /2 1sen(n0 t )d t + 0 1sen(n0 t ))d t


2 0, n = par
n (1 cos(n)) = 4
n = i mpar
n ,
4 P 1 4 si n(30 t ) si n(50 t )
f (t ) = n=i mpar n si n(n0 t ) = si n(0 t ) + 3 + 5 + ...

4
b)
2
R 1
an = t cos(n0 t )d t
1
h2 0
R i
= 2 n si n(nt ) 0 n1 0 si n(nt )d t
1

1
= 2 n 2
(cos(n) 1)


0, n = par
an =
22n 2 , n = i mpar
R
2
b n = 2 0 1 t si n(n0 t )d t

1 1
= n cos(n) = n (1)n

f (t ) = 14 22 cos(t ) + cos(3t
9
)
+ cos(5t
25
)
+ ... 1 si n(t ) + si n(2t
2
)
si n(3t )
3 + ...

c)
(Trem de impulsos):
c k = N1 [n l N ]exp( j k 2 1
[n]exp( j k 2 1
P P+ P
l = N n) = N N n) = N

Portanto:
1
[n kN ] = exp( j k 2
P+ P
k= N N n)

Observe que os coeficientes da serie de fourier sao constantes( isto , de periodo igual
a 1 para qualquer que seja N).

d)
(Pulso impar):
c k = 14 2n=1 ([n + 1] + [n 1]) exp( j k 2
P
4 n)
1 2 j

= 4 exp( j k 2 ) + exp( j k ) = 2 sen(k 2 )
c 0 = 0 (valor mdio), c 1 = j /2, c 2 = 0, c 3 = + j /2 = c 1
x[n] = c 1 exp( j 2 2
4 n) + c 1 exp( j 4 n) = sen( 4 )
2

e)
(Pulso par):
1 P5
c k = 10 n=4 ([n + 1] + [n] + [n 1]) exp( j k 2
10n)
1 2 2
1
10 1 + exp( j k 10 ) + exp( j k 10 = 10 1 + 2cos(k 5)
3 1 P5
c 0 = 10 (valor mdio)= 10 n=4 p[n]
c k,k=0,...,9 = [0.30 0.26 0.16 0.04 0.06 0.10 0.06 0.04 0.16 0.26]

7. Calcule a transformada de Fourier de cada um dos seguintes sinais:


a) [e at cos0 t ]u(t ), a > 0

b) e 3|t | sen2t

1 + cost , |t | 1
c) x(t ) =
0, |t | > 1

5
a k (t kT ), || < 1
P
d) k=0

e) [t e 2t sen4t ]u(t )

Resposta:

a)
e at cos(0 t )u(t ) = 21 e at e j 0 t u(t ) + 12 e at e j 0 t u(t )
X ( j ) = 2(a j 1 0 + j ) + 2(a+ j 1 0 + j )
b)
x(t ) = e 3|t | si n(2t )u(t ) + e 3|t | si n(2t )u(t )
x 1 (t ) = e 3|t | si n(2t )u(t ) X 1 ( j ) = ( j +3+2 j )(2 j +32 j )
2
x 2 (t ) = e 3|t | si n(2t )u(t ) = x 1 (t ) X 2 ( j ) = (32 j + j )(3+2 j + j )
24 j
X ( j ) = X 1 ( j ) + X 2 ( j ) = (13+4+2 )(13+42 )
c)
X ( j ) = 2sin + si n
+
si n

d)
X ( j ) = 1ae1 j T
e)
x(t ) = (1/2 j )t e 2t e j 4t u(t ) (1/2 j )t e 2t e j 4t u(t )
8( j +2)
X ( j ) = ( j +24 j )2 ( j +2+4 j )2

8. Determine o sinal de tempo contnuo correspondente a cada uma das seguintes trans-
formadas
2sen[3(2)]
a) X ( j ) = (2)

b) X ( j ) = cos(4 + /3)

c) X ( j ) = 2[( 1) ( + 1)] + 3[( 2) + ( + 2)]


Resposta:

a)
e j 2t ,

|t | < 3
x(t ) =
0, ot her wi se
b)
x(t ) = 12 e j /3 (t 4) + 12 e j /3 (t + 4)
c)
2j
x(t ) = si nt + 3 cos(2t )

9. a) Calcule a convoluo de cada um dos seguintes pares de sinais x(t) e h(t) calculando
X ( j ) e H ( j ), usando a propriedade de convoluo e fazendo a transformada inversa
i. x(t ) = t e 2t u(t ), h(t ) = e 4t u(t )

6
ii. x(t ) = t e 2t u(t ), h(t ) = t e 4t u(t )
iii. x(t ) = e t u(t ), h(t ) = e t u(t )
b) Suponha que x(t ) = e (t 2) u(t 2) e h(t ) seja como esboado na figura abaixo.
Verifique a propriedade de convoluo para esse par de sinais mostrando que a
transformada de Fourier y(t ) = x(t ) h(t ) igual a H ( j )X ( j )

Resposta:
i) h ih i
Y ( j ) = X ( j )H ( j ) = (2+1j )2 4+1j
(1/4) (1/4) (1/2)
= 4+ j 2+ j + (2+ j )2
y(t ) = 41 e 4t u(t ) 14 e 2t u(t ) + 12 t e 2t u(t )
ii) h ih i
Y ( j ) = X ( j )H ( j ) = (2+1j )2 (4+1j )2
(1/4) (1/4) (1/4) (1/4)
= 2+ j + (2+ j )2
4+ j + (4+ j )2
y(t ) = 14 e 2t u(t ) + 14 t e 2t u(t ) 14 e 4t u(t ) + 14 t e 4t u(t )
iii)
Y (hj ) =iXh ( j )H
i ( j )
1 1
= 1+ j 1 j
1/2 1/2
= 1+ j + 1 j
y(t ) = 12 e |t |
b)
0, t <1
y(t ) = 1 e (t 1) , 1<t 5
(t 5)
e e (t 1) , t >5
j 3
Y ( j ) = 2e (1+sijn(2)
)
h j 2 i j
e e 2si n(2)
= 1+ j

= X ( j )H ( j )

10. A entrada e sada de um sistema LIT estvel e causal esto relacionadas pela equao
diferencial
d 2 y(t ) d y(t )
2
+6 + 8y(t ) = 2x(t )
dt dt

7
a) Encontre a resposta ao impulso desse sistema.

b) Qual a resposta desse sistema se x(t ) = t e 2t u(t )?

c) Repita o item a) para o sistema LIT estvel e causal descrito pela equao:

d 2 y(t ) p d y(t ) d 2 x(t )


2
+ 2 + y(t ) = 2 2x(t )
dt dt dt2

Resposta:

a)
Y ( j ) 2
H ( j ) = X ( j ) = 2 +2 j +8
1 1
H ( j ) = j +2 j +4
2t 4t
h(t ) = e u(t ) e u(t )
b)
1
X ( j ) = (2+ j )2
2 1
Y ( j ) = X ( j )H ( j ) = 2 +2 j +8 (2+ j )2
Y ( j ) = j 1/4 1/2 1
+2 ( j +2)2 + ( j +2)3 j +4
1/4

y(t ) = 14 e 2t u(t ) 12 t e 2t u(t ) + 12 t 2 e 2t u(t ) 14 e 4t u(t )


c)
Y ( j ) 2( 2
H ( j ) = = p 1)
X ( j ) 2 + 2 j +1
p p p p
22 2 j 2+2 2 j
H ( j ) = 2 + p p
2+ j 2
+ p p
2 j 2
j 2
j 2 p
p p p
(1+ j )t / 2
h(t ) = 2(t ) 2(1 + 2 j )e u(t ) 2(1 2 j )e (1 j )t / 2 u(t )

11. Considere o sinal x(t):

a) Encontre a transformada de Fourier X ( j ) de x(t ).

b) Esboce o sinal:

(t 4k)
X
F (t ) = x(t )
k=
.

8
c) Encontre outro sinal g(t) diferente de x(t) e tal que

(t 4k)
X
F (t ) = g (t )
k=
.

d) Argumente que,embora G( j ) seja diferente de X ( j ), G( j k k


2 ) = X ( j 2 ) para to-
dos os k inteiros. Voc nao deve obter explicitamente G( j ) para responder a este
item.

Resposta:
a)
x(t ) = x 1 (t ) x 1 (t )
1, || < 1/2
onde x 1 (t ) =
0, ot her wi se
X 1 ( j ) = 2 si n(/2)
h i2
X ( j ) = X 1 ( j )X 1 ( j ) = 2 si n(/2)

b)

c)

d)
F ( j ) = X ( j ) 2 ( j ( k 2 )) = G( j ) 2 ( j ( k 2 ))
P P
k= k=
Pode ser reescrito como:
F ( j ) = 2 X ( j k/2)( j ( k 2 )) = 2 G( j k/2)( j ( k 2 ))
P P
k= k=
Isso possvel se:
G( j k/2) = X ( j k/2)

9
12. Na figura abaixo, mostrado uma implementao de um filtro passa-faixa usando mo-
dulao senoidal e filtros passa-baixas. Demonstre que a sada y(t) do sistema idn-
tica quela que seria obtida atravs da modulao com portadora exponencial com-
plexa retendo apenas a parte real da sada.

Resposta:
g 1 (t ) a resposta de H1 ( j ) a x(t )cosc t e
g 2 (t ) a resposta de H2 ( j ) a x(t )si nc t
y(t ) = x(t )e j c t = x(t )cosc t + j x(t )si nc t
(t ) = g 1 (t ) + j g 2 (t )
f (t ) = e j c t (t ) = [cosc t j si nc t ][g 1 (t ) + j g 2 (t )]
A parte real de f(t):
g 1 (t )cosc t + g 2 (t )si nc t

13. Esboce a representao por diagrama de blocos na forma direta I e II das equaes
diferenciais e das equaes das diferenas abaixo.

a) y(t ) = x(t ) + 0, 5x 0 (t ) + 3y 0 (t ).
3 5
b) y(t ) + ddt y(t ) 4 dd t y(t ) = x(t ) + 3 dd t x(t ).

c) 3y(t ) = x(t ) 2x 0 (t ) + x 00 (t ) 0, 3x 000 (t ).

d) y[n] = x[n] + 2x[n 1] + 3x[n 2] + 0, 9y[n 1].

e) 2y[n] + y[n 1] 4y[n 3] = x[n] + 3x[n 5].

f) y[n] = x[n] x[n 1] + 2x[n 2] 3x[n 4].

14. Considerando a condio inicial de repouso, encontre as respostas ao impulso dos sis-
temas LIT descritos pelas equaes a seguir (Dica: questes 2.55 e 2.56 do Oppenheim).

10
a) y 00 (t ) + 3y 0 (t ) + 2y(t ) = x(t ).
Resposta:
Condies iniciais: y(0+ ) = 0, y 0 (0+ ) = 1.
Equao homognea: s 2 + 3s + 2 = 0 s = 2; 1.
2t t
Soluo: h(t ) = Ae +Be ,t 0
Aplicando condies iniciais: h(t ) = (e t e 2t )u(t )

b) y 00 (t ) + 2y(t ) + 2y(t ) = x(t ).


Resposta:
Condies iniciais: y(0+ ) = 0, y 0 (0+ ) = 1.
Equao homognea: s 2 + 2s + 2 = 0 s = 1 j .
(1+ j )t (1 j )t
Soluo: h(t ) = Ae +Be =e t
[Ae j t + B e j t ],t 0
Aplicando condies iniciais: h(t ) = (e t sin t )u(t )

c) y[n] 15 y[n 1] = x[n].


Resposta:
A entrada dada por x[n] = [n]. A equao ser:
y[n] = 51 y[n 1] + x[n]
n x[n] 15 y[n 1] y[n]
0 1 0 1
1 0 1 1
2 0 1 1
3 0 1 1
4 0 1 1

0 ,n<0
h[n] =
1 ,n0

d) y[n] y[n 2] = x[n].


Resposta:
A entrada dada por x[n] = [n]. A equao ser:
y[n] = y[n 2] + x[n]
n x[n] y[n 2] y[n]
0 1 0 1
1 0 0 0
2 0 1 1
3 0 0 0
4 0 1 1
5 0 0 0
6 0 1 1

11

0 , n < 0 e n > 0, mpar
h[n] =
1 , n 0, par

e) y[n] y[n 2] = 2x[n] 3x[n 4].


Resposta:
A entrada dada por x[n] = [n]. A equao ser:
y[n] = y[n 2] + 2x[n] 3x[n 4]
n 2x[n] 3x[n 4] y[n 2] y[n]
0 2 0 0 2
1 0 0 0 0
2 0 0 2 2
3 0 0 0 0
4 0 3 2 1
5 0 0 0 0
6 0 0 1 1

2 ,n=0en=2
h[n] = 1 ,n4

0 , caso contrrio.

15. (Computacional) A partir do link http://www.physionet.org/cgi-bin/atm/ATM?


database=nstdb&tool=plot_waveforms, baixe um sinal de ECG em formato mat de
durao de 1 minuto. Em seguida, carregue-o no matlab:

load signal.mat
x = val(1,:)
t = linspace(1,60,length(x));
plot(t,x); A imagem deve ser semelhante a:

12
Em seguida, insira um ruido gaussiano branco com SNR de 10dBw:

y = awgn(x,10,measured);
a) Construa um filtro de mdia movel de duas amostras e convolua com o sinal de
entrada. Plote o sinal resultante da convoluo.

b) Construa um filtro de mdia movel de trs amostras e convolua com o sinal de


entrada. Plote o sinal resultante da convoluo.

c) O que se pode inferir dos resultados de a) e b).

d) Plote a fase e magnetude dos filtros mdia mvel de duas e trs amostras. Discuta
o resultado.

e) Plote as curvas de nvel para a forma generalizada do filtro de mdia mvel, para
M=0,1,...,10 e N=0,1,...,10 e convolua cada uma com o sinal de ECG ruidoso. Dis-
cuta os resultados.

Resultado Esperado:

13
14

Вам также может понравиться